LSAT and Law School Admissions Forum

Get expert LSAT preparation and law school admissions advice from PowerScore Test Preparation.

 parytownson
  • Posts: 11
  • Joined: Feb 12, 2021
|
#89028
I figured that this was a Required Assumption question and that the correct answer was a supporter assumption answer, but I want to make sure I have the classification and diagramming correct:

Premise 1: If retraining of workers is efficient, then it allows companies to meet their own short term needs (RE :arrow: SN). This is the second sentence of the stimulus and the premise of the argument.

Conclusion: If it is a large governmental training program, then retraining of workers is not effective (LGP :arrow: ~RE). This is the last sentence of the stimulus and the conclusion of the argument.

I need to find a way to connect LGP to SN. Well, the contrapositive of premise 1 -- ~SN :arrow: ~RE -- would allow me to connect LGP to RE by having the following premise: If there is a large governmental training program, then it does not allow companies to meet their short term needs. (LGP :arrow: ~SN).

As such, the argument would go:

1a.) RE :arrow: SN
1b.) Contrapositive: ~SN :arrow: ~RE

2.) LGP :arrow: ~SN

Conc: LGP :arrow: ~RE

This missing premise is exactly what answer choice (D) says. And thus, it is the missing premise of the argument. As such, would this classify as a supporter answer as opposed to a defender answer?

Thanks in advance, Powerscore. Ya'll are really helping me nail down this test.
User avatar
 Beatrice Brown
PowerScore Staff
  • PowerScore Staff
  • Posts: 75
  • Joined: Jun 30, 2021
|
#89085
Hi Pary! Great job on this question :)

You're correct that this is an Assumption question and that the correct answer choice for this question is a Supporter since it connects elements from the premise and conclusion.

However, I would diagram the logic a bit differently. The premise tells us that if retraining is efficient, then it allows individual companies to meet their short-term needs (diagrammed as: RE :arrow: STN met). The conclusion then argues that certain programs (large governmental job retraining programs) don't retrain workers efficiently (or, according to our diagram, ~RE).

How can we conclude that a program doesn't retrain workers efficiently? Well, the contrapositive of the premise is that if short-term needs of the company are not met, then the retraining is not efficient (~STN met :arrow: ~RE). So we can conclude that large governmental job retraining programs are not efficient if they do not allow companies to meet their short-term needs! We want to find an answer choice that provides us with this connection, which, as you correctly pointed out, answer choice (D) does. And since this answer choice closes a gap in the stimulus, you're correct that it's a Supporter!

The main difference between how you diagrammed this question and how I explained approaching it is that the last sentence of the stimulus isn't truly a conditional statement. Instead, the conclusion is stating that a sufficient condition doesn't happen, but to make that conclusion, we need to know that the necessary condition doesn't happen (which is what answer choice (D) tells us).

I hope this helps, and let me know if you have any further questions! Great work :)

Get the most out of your LSAT Prep Plus subscription.

Analyze and track your performance with our Testing and Analytics Package.